2025 AMC 8 Problems/Problem 23

Revision as of 19:27, 2 February 2025 by Dr david (talk | contribs)

Problem

How many four-digit numbers have all three of the following properties?

(I) The tens and ones digit are both 9.

(II) The number is 1 less than a perfect square.

(III) The number is the product of exactly two prime numbers.

$\textbf{(A)}\ 0 \qquad \textbf{(B)}\ 1 \qquad \textbf{(C)}\ 2 \qquad \textbf{(D)}\ 3 \qquad \textbf{(E)}\ 4$

Solution

The Condition II perfect square must end in "$00$" because $...99+1=...00$ (Condition I). Four-digit perfect squares ending in "$00$" are ${40, 50, 60, 70, 80, 90}$.

Condition II also says the number is in the form $n^2-1$. By Difference of Squares[1], $n^2-1 = (n+1)(n-1)$. So:

  • $40^2-1 = (39)(41)$
  • $50^2-1 = (49)(51)$
  • $60^2-1 = (59)(61)$
  • $70^2-1 = (69)(71)$
  • $80^2-1 = (79)(81)$
  • $90^2-1 = (89)(91)$

On this list, the only number that is the product of $2$ prime numbers is $60^2-1 = (59)(61)$, so the answer is $\boxed{\text{(B)\ 1}}$.

~Soupboy0

Video Solution 1 by SpreadTheMathLove

https://www.youtube.com/watch?v=jTTcscvcQmI

Video Solution (A Clever Explanation You’ll Get Instantly)

https://youtu.be/VP7g-s8akMY?si=wexxSYnEz2IcjeIb&t=3539 ~hsnacademy

Video Solution by Thinking Feet

https://youtu.be/PKMpTS6b988

Video Solution by Dr. David

https://youtu.be/jn-qIwv57nQ

See Also

2025 AMC 8 (ProblemsAnswer KeyResources)
Preceded by
Problem 22
Followed by
Problem 24
1 2 3 4 5 6 7 8 9 10 11 12 13 14 15 16 17 18 19 20 21 22 23 24 25
All AJHSME/AMC 8 Problems and Solutions

The problems on this page are copyrighted by the Mathematical Association of America's American Mathematics Competitions. AMC logo.png